Difference between revisions of "2013 Mock AIME I Problems/Problem 3"

m (see also box)
Line 6: Line 6:
  
 
Let <math>y=(7-4\sqrt{3})^{2^{2013}}</math>. Notice that <math>y<<1</math> and that, by expanding using the binomial theorem, <math>x+y</math> is an integer because the terms with radicals cancel. Thus, <math>y=1-\{x\}</math>. The desired expression is <math>x\left(1-\{x\}\right)=xy=((7+4\sqrt{3})(7-4\sqrt{3}))^{2^{2013}}=\boxed{1}</math>.
 
Let <math>y=(7-4\sqrt{3})^{2^{2013}}</math>. Notice that <math>y<<1</math> and that, by expanding using the binomial theorem, <math>x+y</math> is an integer because the terms with radicals cancel. Thus, <math>y=1-\{x\}</math>. The desired expression is <math>x\left(1-\{x\}\right)=xy=((7+4\sqrt{3})(7-4\sqrt{3}))^{2^{2013}}=\boxed{1}</math>.
 +
 +
==See also==
 +
* [[2013 Mock AIME I Problems/Problem 2|Preceded by Problem 2]]
 +
* [[2013 Mock AIME I Problems/Problem 4|Followed by Problem 4]]

Revision as of 07:38, 30 July 2024

Problem

Let $\lfloor x\rfloor$ be the greatest integer less than or equal to $x$, and let $\{x\}=x-\lfloor x\rfloor$. If $x=(7+4\sqrt{3})^{2^{2013}}$, compute $x\left(1-\{x\}\right)$.

Solution

Let $y=(7-4\sqrt{3})^{2^{2013}}$. Notice that $y<<1$ and that, by expanding using the binomial theorem, $x+y$ is an integer because the terms with radicals cancel. Thus, $y=1-\{x\}$. The desired expression is $x\left(1-\{x\}\right)=xy=((7+4\sqrt{3})(7-4\sqrt{3}))^{2^{2013}}=\boxed{1}$.

See also